Questions   
Sort: 
 #4
avatar+26376 
+1
Dec 16, 2019
 #3
avatar+26376 
+2

Find the numerical value of \(\sin(40^\circ)*\sin(80^\circ) + \sin(80^\circ)*\sin(160^\circ) + \sin(160^\circ)*\sin(320^\circ)\)

 

\(\begin{array}{|rcll|} \hline && \sin(40^\circ)\sin(80^\circ) + \sin(80^\circ)\sin(160^\circ) + \sin(160^\circ)\mathbf{\sin(320^\circ)} \\\\ && \boxed{ \mathbf{\sin(320^\circ)} = \sin(360^\circ-40^\circ) = -\sin(40^\circ) } \\\\ &=& \sin(40^\circ)\sin(80^\circ) + \sin(80^\circ)\sin(160^\circ) - \sin(160^\circ)\sin(40^\circ) \\ &=& \sin(40^\circ)\sin(120^\circ-40^\circ) + \sin(120^\circ-40^\circ)\sin(120^\circ+40^\circ) - \sin(120^\circ+40^\circ)\sin(40^\circ) \\ &=& \sin(40^\circ)\sin(120^\circ-40^\circ) - \sin(120^\circ+40^\circ)\sin(40^\circ)+ \sin(120^\circ-40^\circ)\sin(120^\circ+40^\circ) \\ &=& \sin(40^\circ)\Big( \underbrace{\sin(120^\circ-40^\circ) - \sin(120^\circ+40^\circ)}_{\boxed{\mathbf{\text{Formula:}~2\cos(x)\sin(x) = \sin(x+y)-\sin(x-y)} \\ \sin(120^\circ-40^\circ) - \sin(120^\circ+40^\circ) = -2\cos(120^\circ)\sin(40^\circ) }} \Big)+ \sin(120^\circ-40^\circ)\sin(120^\circ+40^\circ) \\\\ &=& \sin(40^\circ)\Big( -2\mathbf{\cos(120^\circ)}\sin(40^\circ)\Big)+ \sin(120^\circ-40^\circ)\sin(120^\circ+40^\circ) \\\\ && \boxed{ \mathbf{\cos(120^\circ)} = \cos(180^\circ-60^\circ) = -\cos(60^\circ)=-\dfrac{1}{2} } \\\\ &=& \sin(40^\circ)\Big( -2\left(-\dfrac{1}{2}\right)\sin(40^\circ)\Big)+ \sin(120^\circ-40^\circ)\sin(120^\circ+40^\circ) \\ &=& \sin^2(40^\circ)+\underbrace{\sin(120^\circ-40^\circ)\sin(120^\circ+40^\circ)}_{ \boxed{ \mathbf{\text{Formula:}~-2\sin(x-y)\sin(x+y) = \cos(2x)-\cos(2y)}\\ \sin(120^\circ-40^\circ)\sin(120^\circ+40^\circ)=-\dfrac{1}{2}\Big(\cos(240^\circ)-\cos(80^\circ) \Big) } } \\\\ &=& \sin^2(40^\circ)-\dfrac{1}{2}\Big(\mathbf{\cos(240^\circ)-\cos(80^\circ)} \Big) \\ && \boxed{ \mathbf{\cos(240^\circ)} = \cos(180^\circ+60^\circ) = -\cos(60^\circ)=-\dfrac{1}{2}\\ \cos(80^\circ) = \cos(2*40^\circ) = 2\cos^2(40^\circ)-1 } \\\\ &=& \sin^2(40^\circ)-\dfrac{1}{2} \Bigg( -\dfrac{1}{2} -\Big( 2\cos^2(40^\circ)-1\Big) \Bigg) \\ &=& \sin^2(40^\circ)-\dfrac{1}{2} \Big( -\dfrac{1}{2} - 2\cos^2(40^\circ)+1 \Big) \\ &=& \sin^2(40^\circ)+\dfrac{1}{4} + \cos^2(40^\circ)-\dfrac{1}{2} \\ &=& \underbrace{\sin^2(40^\circ) + \cos^2(40^\circ)}_{=1} +\dfrac{1}{4}-\dfrac{1}{2} \\ &=& 1 +\dfrac{1}{4}-\dfrac{1}{2} \\\\ &=& \dfrac{1}{4}+\dfrac{1}{2} \\\\ &=& \mathbf{ \dfrac{3}{4} } \\ \hline \end{array}\)

 

laugh

Dec 16, 2019
 #1
avatar+26376 
+1

The spherical coordinates of (-3, 4, -12) are \((\rho, \theta, \varphi)\).
Find \(\tan \theta + \tan \varphi\).

 

I assume: the spherical coordinates are \((\text{radius}~ \rho, \text{inclination}~ \theta, \text{azimuth}~ \varphi)\)

\(\begin{array}{|lrcrl|} \hline (1) & x &=& -3 &= \rho \sin(\theta) \cos(\varphi) \\ (2) & y &=& 4 &= \rho \sin(\theta) \sin(\varphi) \\ (3) & z &=& -12 &= \rho \cos(\theta) \\ \hline \end{array} \)

 

\(\begin{array}{|lrcll|} \hline \dfrac{(2)}{(1)}: & \dfrac{\rho \sin(\theta) \sin(\varphi)}{\rho \sin(\theta) \cos(\varphi)} &=& \dfrac{4}{-3} \\ & \mathbf{\tan(\varphi)} &=& \mathbf{-\dfrac{4}{3}} \\ \hline \end{array}\)

\(\begin{array}{|lrcll|} \hline \dfrac{(2)}{(3)}: & \dfrac{\rho \sin(\theta) \sin(\varphi)} {\rho \cos(\theta)} &=& \dfrac{4}{ -12} \\ & \tan(\theta)\sin(\varphi) &=& -\dfrac{1}{3} \quad | \quad \text{square both sides} \\ & \mathbf{\tan^2(\theta)\sin^2(\varphi)} &=& \mathbf{\dfrac{1}{9}} \\ \hline \dfrac{(1)}{(3)}: & \dfrac{\rho \sin(\theta) \cos(\varphi)} {\rho \cos(\theta)} &=& \dfrac{-3}{ -12} \\ & \tan(\theta)\cos(\varphi) &=& \dfrac{1}{4} \quad | \quad \text{square both sides} \\ & \mathbf{\tan^2(\theta)\cos^2(\varphi)} &=& \mathbf{\dfrac{1}{16}} \\ \hline & \tan^2(\theta)\sin^2(\varphi)+\tan^2(\theta)\cos^2(\varphi) &=& \dfrac{1}{9}+\dfrac{1}{16} \\ & \tan^2(\theta)\left(\underbrace{ \sin^2(\varphi)+ \cos^2(\varphi)}_{=1} \right) &=& \dfrac{25}{9*16} \\ & \tan^2(\theta) &=& \dfrac{25}{9*16} \\ & \tan(\theta) &=& \dfrac{5}{3*4} \\ & \mathbf{\tan(\theta)} &=& \mathbf{\dfrac{5}{12}} \\ \hline \end{array}\)

 

\(\begin{array}{|rcll|} \hline \tan(\theta) + \tan(\varphi) &=& \dfrac{5}{12} -\dfrac{4}{3} \\\\ &=& \dfrac{15-48}{36} \\\\ &=& \dfrac{-33}{36} \\\\ \mathbf{\tan(\theta) + \tan(\varphi)} &=& \mathbf{-\dfrac{11}{12}} \\ \hline \end{array}\)

 

laugh

Dec 16, 2019

5 Online Users

avatar
avatar